Math, asked by musakhan56, 1 year ago

answer this bar graph plzzzz

Attachments:

Answers

Answered by gunjan010773p2afz3
2
u have answered all questions

musakhan56: the ans is corrrect or not
gunjan010773p2afz3: its correct
musakhan56: ok thanku
gunjan010773p2afz3: ur welcome
Answered by kanishkkandee
1
all are correct so send it as correct

Ubhay: 2and4 is wrong
musakhan56: so the answer is
Similar questions